Você está na página 1de 82

1

Carnot cycle
Consider a classical ideal monoatomic gas with N atoms.

(a) Write down the expression of the internal energy.


(b) What is the work done during an isothermal expansion from initial volume
Vi to final volume Vf at temperature T ?
(c) What is the work done during an adiabatic expansion? Use the first law of
thermodynamics.
Assume the gas is the working fluid in a Carnot cycle. In this process, the
gas is expanded at constant T = Th from V1 to V2 . Then it is expanded
at constant entropy from V2 to V3 . At this point the temperature is Tc .
It is then compressed isothermally at T = Tc to a volume V4 , such that it
can be compressed at constant entropy back to V1 and Th .
(d) Compute the work done on each stage of this cycle using reservoirs at Th
and Tc . Obtain the heat exchange at each stage and use the first law of
thermodynamics.
(e) What is the Carnot efficiency for this process (as a function only of temperature Tc and Th )?
Solution
(a) U = N kB T . Function of state only depends on T .
(b) Isothermal expansion:
Z

Vf

W =

dV P = nRT ln
Vi

Vf
.
Vi

(c) Adiabatic expansion:


P V = const or TV1 = const0 , =

5
3

3
Q = 0 U + W = 0 W = U = nR(Tf Ti )
2
(d)
1 2 : isothermal, U = 0, Qh = W = nRTh ln

V2
V1

3
2 3 : adiabatic, Q = 0, W = U = nR(Tc Th )
2
V4
3 4 : isothermal, U = 0, Qc = W = nRTc ln
V3
3
4 1 : adiabatic, Q = 0, W = U = nR(Th Tc )
2
V2
V3
Total amount of work :W = Qh |Qc | = nR[Th ln
Tc ln
]
V1
V4
Problem 2 from Qualifying exam 2010-Spring.

(e)
=

|Qc |
W
Tc ln
=1
=1
Qh
Qh
Th ln

V3
V4
V2
V1

Th V21 = Tc V31 , Th V11 = Tc V41


V3
V2
V3
Tc
V2
=
=1
( )1 = ( )1
V1
V4
V1
V4
Th

Lithium

The ground level of the neutral lithium atom is doubly degenerate (that is, d0
= 2). The first excited level is 6-fold degenerate (d1 = 6) and is at an energy
1.2 eV above the ground level.
(a) In the outer atmosphere of the Sun, which is at a temperature of about
6000 K, what fraction of the neutral lithium is in the excited level? Since
all the other levels of Li are at a much higher energy, it is safe to assume
that they are not significantly occupied.
(b) Find the average energy of Li atom at temperature T (again, consider only
the ground state and the first excited level).
(c) Find the contribution of these levels to the specific heat per mole, CV , and
sketch CV as a function of T .
Use 1mev = 11.6K.
Solution
(a) If the ground level energy is defined as zero and E is the energy of excited level:
X
Z=
di exp(i ) = 2 + 6 exp(E)
i

The probability that the atom is in its excited level:


P (E) = 6 exp(E)/Z = 6 exp(E)/[2 + 6 exp(E)] = 3/[3 + exp(E)]
Since E = 1.2eV , T = 6000K( 0.5eV ), E = 2.32, exp(E) 10, we get:
P (E) = 3/(3 + 10) = 0.23.
(b) The average energy per atom is:
hi =

6e
3
1 Z
=
=
Z
2 + 6e
e +3

(c) The specific heat is:


Cv = [

hi
3kB ()2 e
]V =
T
(e + 3)2

lim Cv = lim Cv =

T 0

32 /kB T
e
kB T 2

lim Cv = lim Cv =

Problem 11 from Qualifying exam 2010-Spring.

32
16kB T 2

Probability of being void

A room of volume V = 10m3 is under standard conditions (classical ideal gas


at pressure 1 atm and temperature T = 273K).
(a) Estimate the probability that at any instant of time a small subvolume v0
of V becomes totally void, because of spontaneous statistical fluctuation,
assuming v0  V .
(b) Specialize to the following three subvolumes of the room: (i) v0 = V /N ,
where N is the number of particles, (ii) 1cm3 and (iii) 1A3 (recall 1A=1010 m).
You can use the following approximate values: 1 atm=100kPa=100kN/m2 , and
k = 1.38 1023 J/K for the Boltzmann constant.
Solution
(a) The probability that particle 1 is NOT in v0 is
v0
.
V

P1 = 1

Therefore, the probability that all N particles are not in v0 is


PN = (1

v0 N
)
V

For v0 /V small,
PN = exp(

N v0
).
V

(b) (i) PN = e1 = 0.367879...for v0 = V/N.


(ii) P V = N kT implies
N = P V /T k =

105 10
2.65 1026 .
273 1.38 1023

For v0 = (0.01)3 m3 = 1.0 106 m3 we have v0 /V = 107 and


PN = exp(2.65 1019 ) = 0.
(iii) For v0 = 1A3 = 1030 m3 we have v0 /V = 1031 and
PN = exp(2.65 105 ) = 1 2.65 105 = 0.999973 1.

Problem 5 from Qualifying exam 2011-Fall.

Heating in house
Does heating in winter

(a) change the energy content of the house?


(b) change the entropy content of the house?
Answer these questions in an ideal gas approximation with u = cv T for the oneparticle internal energy and cv constant. Compare states of constant pressure
at an initial and final temperature assuming that the initial and final states are
each in equilibrium.
Solution
(a) No. Using Ti for the initial temperature and the ideal gas equation pV = nRT ,
the initial internal energy becomes
Ui = N cv Ti =

cv
pVi .
kB

Heating at constant pressure to a temperature Tf > Ti results in a final volume


Vf given by pVf = nRTf . For this volume the energy is
Uf =

cv
pVf .
kB

However, the house does not expand, so that the energy in the house is
Uf,house =

Vi
Uf = Ui .
Vf

As the volume of the house stays constant the energy inside the house is in the
final state the same as in the initial one.
(b) For the entropy of an ideal gas at constant pressure we take the equation
S = N cv ln

T
V
T
+ N kB ln
+ S0 = N (cv + kB ) ln
+ S0 ,
T0
V0
T0

where T > T0 and T0 is the liquefaction temperature. We neglect in the following


the residual entropy of the liquid. The entropy in the house is then


PV
T
cv + kB
S(T ) '

ln
kB
T
T0




cv + kB
PV
T0
T
=

ln
.
kB
T0
T
T0
Now, it is easy to see that the function ln x/x increases monotonically for 0 <
x < e = 2.718..., decreases monotonically x > e. Therefore entropy changes by
heating in the winter. If we use T0 = 78K, which is the actual boiling point of
air, even at T = 233K = 40 C, T /T0 > e. So it actually decreases.

Problem 6 from Qualifying exam 2011-Spring.

Bose-Einstein condensation

In this problem, we will be exploring the Bose-Einstein transition temperature,


TBE , of a gas of N non-interacting spin-less Bose particles. Below TBE there is
a macroscopic occupation of the ground state. Here, each of the particles has
mass m and all are enclosed in a three-dimensional volume V at temperature
T.
(a) Find an expression for the density of available single-particle states D()
as a function of the single-particle energy . Sketch D()
(b) What is the allowed range of the chemical potential for a non-interacting
Bose-Einstein gas?
(c) Write down an expression for the mean occupation number of a single
particle state, < n >, as a function of , T, and (T ), where TBE < T < .
Sketch < n > as a function of .
(d) Write down an integral expression which implicitly determines (T ). Put
the integral into dimensionless form.
(e) Using the result from part (d), find TBE .
Solution
(a) We assume these particles are non-relativisic, and therefore have energy dispersion
2 2
k
. The single-particle density of states is given by:
(k) = h2m
Z
D() = V

d3 k
2 k2
h
)
(
3
(2)
2m

We can use a change of variables:


r
2 k2
h
2mx
k=
2m
h2

r
r
h2

h2 2mx

2
h2 x
dx =
kdk =
2 dk =
m
m
m
h

r
m
dk =
dx
2
h2 x
r
Z
4V
2mxdx
m
D() =
( x)
2
(2)3
h

2
h2 x

3/2

V
2m
D() =

(2)2
h

x=

The plot goes like the square-root of . The factors in front scale the curve up
or down, but do not change its shape.
Problem SM7 from FSU Qualifying Exam Wiki.

(b) Recall that z = e . For a Bose gas, the occupation factor is:
n() =

1
z 1 e 1

The occupation factor is always greater than/equal to zero (it makes no sense
to have negative occupancy). When  = 0 the occuption factor becomes:
n(0) =

z
1
=
0
z 1 1
1z

This inequality can only be satisfied if z < 1. Since z = e , we find that < 0.
(c) For a Bose-Einstein gas, the occupation number is given by:
n() =

1
1
=

z 1 e 1
e kT 1

As  the distribution diverges, and as  the distribution goes to zero:

(d) The particle density can be written as a sum over the occupation number for
all possible energies, which in turn must be broken down into the macroscopic
occupation of the ground state plus all other energy states:
Z
Z
n(0)
N
1 X
d3 k
1 z
d3 k
1
=
n((k)) =
+
n()
=
+
3
V
V
V
(2)
V 1z
(2)3 z 1 e 1
k

Since z = e , when this expression is evaluated we could solve for z and then
find . At T = TBE , z 1 ( 0), so we have:
Z
N
d3 k
1
=
2 k2
V
(2)3 eBE h2m
1
We can do a change of variables similar to the first part of the problem:
x=

BE
h2 k 2
2mx
k2 =
2m
BE
h2

1/2
2m
1
dx
x
BE
h2

1/2
Z
N
4
2mx 1
2m
1 1
=
dx
V
(2)3
x ex 1
BE
h2 2 BE
h2
3/2 Z

N
x
1
2m
=
dx
V
(2)2 BE
ex 1
h2
dk =

(e)

1
2

x
dx
ex 1

3/2 

N
1
2m
1/2
=
1.306
2
V
(2)2 BE
h


3/2
(2)2 N
h2
1
= (kTBE )3/2
V
2m
1.306 1/2
!2/3
 2 3/2
(2)2 N
h

1
TBE =
V
2mk
1.306 1/2
1
N
=
V
(2)2

2m
BE
h2

3/2 Z

Two questions

Question One:
A sealed, perfectly insulated container is partitioned into two halves. One contains an ideal gas at temperature T and the other, a vacuum. The partition is
broken instantly and the gas expands to fill the entire container.
(a) After the gas reaches equilibrium, is the temperature great than, less than,
or equal to T? Explain.
(b) Would your answer be different if the gas were real? Explain.
Question Two:
The mean free path is defined as the average distance traveled by a molecule
between two consecutive collisions with the other molecules. Assume that the
absolute temperature of air increases by a factor of two, while the air pressure
remains the same. Does the mean free path change? If yes, by how much? If
not, why?
Solution
Question One:
(a) The temperature is unchanged because it is an intensive quantity and it is unchanged by the size of the container.
(b) Real gases have molecules that interact, so a real gas in a large container would
have molecules that interact less. The increase in volume also causes a increase
in the potential energy and since it is an insulated container that means the
kinetic energy must decrease to keep the energy constant thus resulting in a
decrease in temperature.
Question Two:
The mean free path, , is proportional to temperature. This makes sense if you think
about the relationship between pressure, volume, and temperature, PV T. The mean
free path is proportional to volume in this situation since the more volume there is
for the same number of particles to interact means that there will be fewer collisions.
Therefore one can think about the mean free path as being proportional to pressure, ie
T/V. Therefore if the temperature increase by a factor of two while the pressure
remains the same, the mean free path will increase by a factor of two as well.

Problem SM10 from FSU Qualifying Exam Wiki.

Pressure on a solid

Consider a solid. At zero temperature, the solid has the following dependence
of pressure on the chemical potential,
P0 () = a( 0 ) + b( 0 )2
Now make the temperature nonzero. According to the quantum theory of solids
constructed by Debye, at nonzero, but small, temperature T the pressure is
the sum of the zero-temperature pressure, P0 (), and the pressure of a gas
of quasiparticles called phonons. Phonons are quanta of sound; they behave
like bosons with three possible polarizations. The energy of a phonon with
momentum p is:
 = u|p|
where u is the speed of sound, which can be regarded as a constant in dependent
of the chemical potential .
(a) From the information above, compute the pressure at low temperatures,
P (T, ). Note that the chemical potential of phonons is equal to zero.
(b) The solid is kept at zero pressure. Assuming the temperature is small, find
the chemical potential as a function of temperature.
(c) Find the number density of atoms as a function of temperature, fixing the
pressure to zero. Find the thermal expansion coefficient at zero pressure.
(d) Using general arguments, including thermodynamic stability, show that the
coefficients a and b appearing in the expression for P0 are both positive,
as is the thermal expansion coefficient computer in part (c).
Solution
(a) Pressure is related to the grand potential, , by the the relationP
P V = , where
N
= kT ln(Z) and Z is the grand partition function Z =
N =0 z QN . For
bosons the grand partition function is given by:
Z=

Y
k

Y
1
1

1 ek
1 e(k u)
k

= kT ln Z = kT

ln(1 ek )

We can turn the sum on k into an integral using the following relation:
Z
1 X
d3 k
=
V
(2)3
k

Z
= kT V

d3 k
ln(1 ek )
(2)3

Problem SM12 from FSU Qualifying Exam Wiki.

k = u|p| = u
hk
Z
kT V 4
=
k2 dk ln(1 euhk )
(2)3
Make the substitution = u
hk to turn the integral into a unitless quantity:

3 Z
kT V 4
1
=
2 d ln(1 e )
(2)3

hu
= (kT )4 V

4
(2)3

1
hu

3 Z

2 d ln(1 e )

We can group all of the constants together can call it and that simplifies the
equation to:
= T 4V
P V = = (T )4 V
P = T 4
So the total pressure is:
Ptot = P0 + P = a( 0 ) + b( 0 )2 T 4
(b) If the total pressure is kept at zero, we can write:
0 = a( 0 ) + b( 0 )2 T 4
Solving this quadratic equation give us:
( 0 ) =

a2 + 4bT
2b

p
a2 + 4bT
+ 0
2b
This can be simplified slightly by expanding the square-root, since we are assuming T is small (were not given any information about a or b, so we must
just assume theyre both much larger than T):
q
a a 1 + 4bT
a2
=
+ 0
2b
=

a a(1 +
2b

2bT
a2

+ 0

We have the two solutions:


=

( 2bT
)
T
a
+ 0 =
+ 0
2b
a

)
a + ( bT
)
2a ( 2bT
a
a
+ 0 =
+ 0
2b
b
We will only consider the first solution because cannot be less than 0 and
we are assuming that all of the constants are positive ( is positive and the
explanation for the sign of a and b *should* be provided in part (d).)
=

(c) The number density is given by n =


found by:

N
V

. The mean occupation number N can be




N =
Another expression for N is:
X
X
N=
n() =
k

d
d

X
1
1
=
() 1
z 1 e 1
e
k

Substituting in our expression for at zero pressure and turning the sum into
an integral:
Z
Z
d3 k
d3 k
1
1
N =V
=
V
 0
T
(2)3 e( a 0 ) 1
(2)3 e kT
ka
kT 1
Z
Z
d3 k
d3 k
1
1
N =V
=V
0


3
(2) e ka e( kT kT ) 1
(2)3 e ka e(0 ) 1
+ 0
= ( 0 ) = (u
hk 0 ) = u
hk 0 k =
u
h
d
d = u
hdk dk =
u
h
2
Z 
+ 0
d
1
4V
N=

(2)3
u
h
u
h e ka e 1
Z

4V
1
N=
2 + 20 + 2 20 d
(2u
h) 3
e ka e 1
Z

Z
Z
1
1
1
4V
2
2 2
d

d
+
2
+

d
N=
0

0
(2u
h) 3
e ka e 1
e ka e 1
e ka e 1
Since each of these integrals is a dimensionless constant, call them A, B, C, we
have:

4V
N=
A + 20 B + 2 20 C
(2u
h)3


4V k3
T2
T 2
3
N=
T
A
+
2
B
+

C
0
0
(2u
h) 3
k
k2
So, finally, we have the number density as a function of temperature:


N
4k3
T2
T 2
3
=
T
A
+
2
B
+

C
0
0
V
(2u
h) 3
k
k2

The coefficient of thermal expansion describes how the size of an object changes
with temperature:
=

1
V

dV
dT


1
T2
T
T 3A +
20 B + 2 20 C
k
k

2 

3
2
N (2u)
dV
T
T 2
T 40 B
1 2
3
2
=
T A+
20 B + 2 0 C
3T A +
+ 2 0 C
dT
4k3
k
k
k
k
V =

N (2u)3
4k3

(d) We dont have much of an explanation for this. Looking at the simplified expression for at zero pressure suggests a must be positive because the minimum of
is 0 .

Chain-like molecule

A chain-like molecule is composed of N rigid straight line segments connected


by hinges that can only be in two states, namely at an angle = 0 or . The
figure shows a typical configuration (but with the = angles slightly smaller
than 180 for graphical clarity). Each rigid line segment carries a magnetic
moment aligned with the segment, and the magnetic field points in the xdirection, B = B ex . Since all line segments point in the x-direction, all magnetic
moments point either along or opposite to the magnetic field.
(a) Calculate the entropy S(N,T) of the chain.
(b) Calculate the average distance between the end points of the chain.
(c) Calculate the root-mean square fluctuation in the endpoint separation.
(d) Discuss your result in part (c) in the limit of very high temperatures from
the perspective of random walks. In case you did not succeed in solving
part (c), you should still be able to predict how D(N ) depends on N
using random walk arguments.
Solution
(a) If we have a segments at the total quantity of possible states will be:
N!
=
a!(N a)!

(1)

Knowing that the energy of a magnetic moment is given by E = B, the


energy of the system will be:
E

(N 2a)B

(2)

E
N
+
2B
2

(3)

= a =

From the relation for the entropy S = kB ln and from (1),(3):






N
E
N
E
S = kB ln N ! kB ln
+
! kB ln

!
2
2B
2
2B
Using the Stirlings approximation (ln N ! = N ln N N + O(ln N )) we have:

 



N
E
N
E
kB N ln N kB N kB

ln

1
2
2B
2
2B

 


N
E
N
E
kB
+
ln
+
1
2
2B
2
2B


 

1
E
1
E
kB N
+
ln
+
2
2BN
2
2BN
 


1
E
1
E
+

ln
2
2BN
2
2BN

Problem SM16 from FSU Qualifying Exam Wiki.

(4)

Using the relation

1
T

S
E N

and the previous one for the entropy we have:


!
E
B N
1
kB
=
ln
E
T
2B
B + N

Solving for E:
E = N B tanh

B
kB T

(5)

From (4) and (5) we can have S(N, T )


(b) If the distance between each two consecutive points is l, then the average distace
will be:
hDi = (N 2a)l
(6)
From (2) and (6):
E=

hDi
B
l

(7)

And from (5) and (7) we have:




B
hDi = N B tanh
l
kB T
This result can also be obtain from :
P
(N 2a)leE
E
hDi =
Z
P E
Ee
l E
=
B
Z
l
hEi
=
B


l
B
=
N B tanh
B
kB T


B
l
=
N B tanh
kB T
(c)

D2

=
=


D2 hDi2
P
(N 2a)2 l2 eE

Z
2

=
=

l
2 B 2

E 2 eE

P
2
(N 2a)leE
E

P E 2
Ee
l E

B
Z


l2 
2
E hEi2
2 B 2


l2
E

2 B 2

(8)

And from (5) and (8) we have:

D2

l2

(N B tanh B)



B
l2
2 2
2
N

B
Sech
2 B 2
kB T

2 B 2

= D(N, T ) =


N lSech

B
kB T

(d) From part (c) we have:


D(N, T ) =

N
cosh

B
kB T

l


B
kB T

1, therefore:

D(N, T ) l N

For very high temperatures cosh

1D ideal gas

A one-dimensional monatomic ideal gas with N atoms at temperature T is


confined to a line of length L. (An ideal gas has no collisions, so the atoms
can move through each other.) Because the atoms are in motion they exert an
effective (time averaged) force F on each end of the line.
(a) Find F and show that F L is related to the average energy of the atoms
(b) Use the equipartition theorem to find an expression for the heat capacity
at constant length, CL
(c) Find the equation of state (i.e., show how to write FL in terms of the
temperature T).
Solution
(a) Method 1:
Find the force by finding the change in momentum:
F =
pi = mvi

p
t

pf = mvi

p = 2mv
2L
x = vt t =
v
mv 2
p
=
F =
t
L
N mv 2
1
Ftotal =
Etotal = N mv 2
L
2
F L = N mv 2 = 2Etotal

Method 2: (Must do parts (b) and (c) first)


P = F (in one dimension). From part (b): E = 12 N kT . From part (c): F L =
N kT
F L = nKT = 2E
(b) The equipartition theorem gives the following relationship (where f is the number
of degrees of freedom, in this case f = 1):
E=

1
1
f N kT = N kT
2
2

The heat capacity at constant length is given by:




E
1
CL =
= Nk
T L
2
Problem SM17 from FSU Qualifying Exam Wiki.

(c) Method 1:
We found an expression for the energy in part (A) using the kinetic energy of
the particles. We also found an expression for the energy in part (B) using the
equipartition theorem. Equating these two expressions for the energy:
1
1
N kT = N mv 2
2
2

E=
From part (A) we know that:

F L = N mv 2 mv 2 =

F L
N

This gives us:


N kT = F L
Method 2:
Q1 =

X
k

h2 k 2
2m

Z
=L

2 k2
dk h2m
e
2

h2 k 2

h2 k
x=
dx =
dk
2m
m
r
Z
r
m
dx
m
L
L
ex =
Q1 =
2 2
2
x
h2
2
h2
1/2

mkT
Q1 = L
2h2
1
QN =
(Q1 )N
N!
A = kT ln(QN ) = kT [N ln(Q1 ) ln(N !)] = kT [N ln(Q1 ) + N ln(N ) N ]
 


N
A = N kT [ ln(Q1 ) + ln(N ) 1] = N kT ln
1
Q1
"
#
1!








2
N
2h2
N
1
2h2
A = N kT ln
1 = N kT ln
+ ln
1
L mkT
L
2
mkT
A
N kT
=
P L = F L = N kT
L
L
This method is much more complicated and time consuming than method 1.
However if other thermodynamic properties are asked for it may be necessary
to compute A.
P =

10

2-level system

Consider a 2-level system with energy states and + ( 0). Assume


the absolute temperature is T.
(a) Compute the partition function.
(b) Determine the free energy.
(c) Derive an expression for the specific heat C(T ). What are the low-T and
high-T limits of this expression? Sketch your result.
Solution
(a) Partition function:
Z=

e = e + e(+)

(9)

(b) Free energy:


F = kT ln Z = kT ln(e + e(+) )

(10)

where = 1/(kT ).
(c) To calculate the specific heat, you first need the internal energy:
U =

e + ( + )e(+)

ln Z
=
= + /kT

e + e(+)
e
+1

(11)

then
C(T ) =

2 e/kT
U
=
2
2
T
kT (1 + e/kT )

Problem 12 from Qualifying exam 2007-spring.

(12)

Molecules in a plane
Molecules confined to rotate in a plane have quantized energies
En = (
h2 /2I)n2

(n = 0, 1, 2, ...)

(I is the moment of inertia) The levels with n 1 are doubly degenerate.


(a) Compute the rotational partition function in the low- and high-temperature
limits.
(b) Do a classical calculation of the partition function and compare with the
results in part (a).
(c) Compute the specific heat per molecule in the low- and high-temperature
limits.
Solution
(a)
En

2 2
h
n
2I
X
X h 2 n2
gn eEn = 1 + 2
e 2I
n

(1)
(2)

n=1

Low temperature:  1, sum over the first few terms,


Z = 1 + 2e

h2
2I

+ 2e

4
h2
2I

+ ...

(3)

High temperature:  1, we can use integral to estimate the summation:


Z
h
2 2
Z = 2
dxe 2I x 1
(4)
0
s
1 2I
= 2
1
(5)
2
h2
s
2I

(6)

h2
(b) Classical:
Z

=
=
=

1
h

p2

ddP e 2I

(7)

p
2
dP e 2I
h
s
2I

h2

(8)
(9)

The classical result is the same as the high temperature approximation, which
makes sense.
Problem 4 from UCLA 1987Fall.

(c)
cV =

U
ln Z
| ,U =
.
T V

(10)

For low temperature,


Z

1 + 2e

h2
2I

(11)
2

ln Z
U

h

2I

ln(1 + 2e

h2

1
ln Z
=

h2

I e
2I + 2

U
h2

1
e 2IkT
= ( )2
h
2
T
I 2kT 2 (2 + e 2IkT
)2

(12)
(13)
h
2

cV

h
2
2 2 1 2IkT
h
.
)
e
I 2kT 2

(14)
(15)

For high temperature,


ln Z

cV

1 2I
ln
2
h2
1
kT
ln Z
=
=

2
2
U
k
=
T
2

(16)
(17)
(18)

Ideal gas

(a) Show from statistical mechanics that


P = nkT
holds exactly for a classical non-interacting gas whether it is relativistic
or not.
(b) Evaluate the energy density (u) of the gas in the extreme relativistic limit
and show from part (a), the relation between P and u.
Hint: pressure is computed from the thermodynamic relation P = F/V ,
where F = kT ln Z; here Z is the many-particle partition function.
Solution
(a) For a non-interacting gas,
Zn =

eEs N =

= Z1N

where
Z1 =

eEs =

1
h3

d3 xd3 peE =

V
h3

d3 peE .

Then
ln Z1 = ln V + function independent on V,
so
P =

N ln Z1
N
=
= nkT.
V
V

(b) In the extreme relativistic limit,


= pc,
the partition function is then
Z1

Z
1
e = 3
d3 xd3 pe
h
Z
V
8V
4
,
dpp2 epc =
3
h3
(hc)
0

=
the internal energy is

U =N

ln Z1
3N
=
= 3N kT,
()

the relation between P and u is


P = nkT =

Problem 16 from UCLA 1987Fall.

1U
1
N kT
=
= u.
V
3V
3

Heat engine

A heat engine operates by extracting energy from a container of gas consisting


of N atoms with constant specific heat C per atom, initially at temperature T .
The engine releases heat into the atmosphere which is at temperature T0 < T .
Find the maximum work which can be extracted from the gas.
Solution
The maximum work occurs when the total entropy change is zero, Stotal = 0, i.e.
Z T0
Q0
N CdT
+
=0
T
T0
T
where the first term is the entropy change of gas in the container and the second
term is the entropy change of the atmosphere, with Q0 being the heat released to the
atmosphere. So
T
Q0 = T0 N C ln
,
T0
then
Z T
T
Wmax = Q Q0 =
CN dT Q0 = N C(T T0 ) N CT0 ln
T0
T0

Problem 6 from UCLA 1989Spring.

Quantum oscillator

(a) What is the average energy u of a quantum oscillator of angular frequency


in thermal equilibrium at temperature T ?
(b) What is the expectation value for the fluctuations

(uu)2
?
u2

(c) In the Einstein model of a crystal each ion has 3 degrees of freedom with
the same frequency . Calculate the specific heat of the crystal.
Solution
(a) The partition function is
Z=

e(n+ 2 )h = (e

h
2

h
2

)1

n=0

u=

ln Z =

(e 2
2

h
2

+ e

h
2

h
2

h
coth
2
2

(b)
(u u)2 = u2 u2
P
u2

n=0 [(n

so
u2 u2 =

+ 12 )
h]2 e(n+ 2 )h
=
Z

2
Z
2

(
h)2
u
h

h
=
(coth
) = h

2
2
(e 2 e 2 )2

u2 u2
4
h

= h
.
= sech2

2kT
u2
(e 2 + e 2 )2
(c) Consider N ions crystal,
CV = 3N

Problem 10 from UCLA 1989Spring.

h
2
3N k( kT
)
u
=
h

T
(e 2kT e 2kT )2

Gravitation

The system shown in the diagram is thermally isolated at a temperature, T ,


in a gravitational field of strength, g, and has a total heat capacity, C. The ball
is initially at a height, h, above its rest position, as shown. Then the ball is
released, and a new equilibrium is reached. (Assume that M gh  kT .)

(a) What is the increase, T , in the temperature of the system?


(b) What is the change in entropy of the system?
(c) What is the probability, P , that the ball will spontaneously return to its
original position?
(d) Evaluate P for T /T  1, M gh = 105 ergs, and T = 300K.
Solution
(a) After new equilibrium is reached,
M gh = CT T =

M gh
C

The gravitational potential goes into the heat due to collisions between the ball
and the gas molecules.
(b)
T dS = dQ = CdT
T + T
M gh
CdT
= C ln
= C ln(1 +
)
T
T
CT

Z
S =
(c)
P = e

S
k

= e k

gh
ln(1+ M
)
CT

=(

1
1+

M gh
CT

)C/k

(d) If /T  1,
C

P = e k

ln(1+ T
)
T

C T
T

e k

C M gh
CT

= e k

= e

M gh
kT

With M gh = 105 ergs, T = 300K,


P e10

/(1.381023 300)

Problem 6 from UCLA 1990Spring.

= e2.510

18

1018 0

Magnetic moments

Consider a solid composed of atoms with magnetic moment per atom.


Assume the moments interact weakly with the lattice vibrations in the solid but
not with each other. In an applied magnetic field H a moment has energy  =
H ( = +H) if it points parallel (antiparallel) to the field, and we assume
that no other orientations are possible. The system is thermally insulated,
initially in equilibrium at temperature T with applied field H, and the field is
suddenly switched from H to H.
(a) Find the new temperature of the system, T 0 , after equilibrium has been
reached, in terms of the given quantities and the specific heat per atom c
due to lattice vibrations. You may assume |T 0 T |  T . Is T 0 larger or
smaller than T ?
(b) Give a numerical estimate as for which initial temperature range the change
in temperature in this process will be larger than 1% for H = 105 gauss.
Assume = B (Bohr magneton). B = 0.927 1020 ergs/gauss, kB =
1.38 1016 ergs/K.
Solution
(a) For temperature T , applied field H, the number of atoms whose magnetic moments are parallel to H is (equilibrium state)
N1 = N

eH
eH + eH

For temperature T 0 , applied field H, the number of atoms whose magnetic


moments are parallel to H is (equilibrium state)
0

N10 = N

e H
+ e 0 H

0 H

The energy for getting the atoms to equilibrium is


(N1 N10 )2H
Since the system is thermally isolated,
(N1 N10 )2H = N C(T T 0 )
0

2H(

2H(

eH
e H
0 H
) = C(T T 0 )
H
H
e
+e
e
+ e 0 H
|T T 0 |  T 0

eH eH
) = C(T T 0 ) = 2H tanh H
eH + eH
tanh H
T 0 = T + 2H
C
T < T0

Problem 7 from UCLA 1990Fall.

(b)
T
> 0.01
T
2H

tanh H > 0.01


CT
2k

H tanh H > 0.01


C
Choose C 3k,
H tanh H > 0.015
Let x = H,
f (x) = x tanh x
4
f 0 (x) = x
x + tanh x > 0
(e + ex )2
We need to find f (x0 ) = 0.015, then for x > x0 , f (x0 ) > 0.015,
x0 tanh x0 = 0.015
Suppose x  1 (proved by result)
x20 = 0.015
x0 = 0.12
so H > 0.12, the initial range of T is T <

H
0.12k

= 56K.

Bose condense

Consider a system of N noninteracting free fermions of mass m and spin 1/2


in a box of volume V at zero temperature. The system is thermally insulated.
Assume that suddenly the fermions turn into bosons of spin 0, with no change
in their mass. Will the system Bose-condense?
Hints:
Z
x1/2
dx x
= 2.32
e 1
0
Z
x3/2
dx x
= 1.78
e 1
0
Density of states per spin for particles of mass m in box of volume V :
g() =

V 2m 3/2 1/2
(
) 
4 2 h2

Solution
For fermions with spin 1/2,
EF

Z
2

g()d = N
0

V 2m 3/2 2 3/2
=N
E
( 2)
4 2
3 F
h

Let

V 2m 3/2
( 2)
= k,
4 2
h

then

2 3 2 N 2/3
h
N 3 2/3
(
)
=(
)
2m
V
k 4

EF =

EF

Z
E

g()d = 2

V 2m 3/2 2 5/2
( 2)
E
4 2
5 F
h

3
N 3 2/3
N 5/3
N(
)
= 0.495 2/3
5
k 4
k

The system is insulated, so the total energy for bosons is also E. For Bose-condense,
= 0,
Z
1
N =
g()d 
e 1
0
Z 1/2

= k0
d
 1
e
0
=

Problem 14 from UCLA 1990Fall.

k0 3/2 2.32
N 1 2/3
( 0
)
k 2.32

Eb0

Z
=

g()d
0

=
=
=
Let Eb0 = E,

1
e 1

k0 5/2 1.78
N 1 5/3
)
1.78
k0 ( 0
k 2.32
N 5/3
0.4387
k02/3

k0 < k V 0 < V

That is the critical volume for Bose-condense is smaller than the volume now. So no
Bose-condensation occurs.

Fiber modes

A thin fiber of length L is stretched between two supports. The speed of


propagation of transverse waves on the fiber is c for both polarizations.
(a) What is the contribution of these modes to the heat capacity of the fiber
at low temperatures, assuming hc/L  kT ? Leave any integral in dimensionless form to display the temperature dependence explicitly.
(b) What is the heat capacity for hc/L  kT ?

Solution
The modes are kn = n/L, n = 1, 2, 3, .... They satisfy Bose statistics.
(a)
E=2

= kc
X
hn

ehn /kT 1

For h
c/L  kT ,
E

=
=
=

CV

Z
2L
h

dk h /kT
0
e
1
Z
2L
h

d h /kT
c 0
e
1
Z
2L kT 2 xdx
h(

)
c
h

ex 1
Z 0
xdx
2L
(kT )2
x 1

hc
e
0
E
4Lk2
=
T
T

hc

(b)
E

X
n

hc
n
L

X
n

Problem 9 from UCLA 1991Spring.

2
hn
eh n /kT 1

hc n/kT
L

If

h
c
L

 kT ,
E

CV

=
=

X
hc
n
hc n
e L /kT
L
n=1

hc hc/LkT
e
L
E
2
hc
hc 1 hc/LkT
=
e
T
L Lk T 2

hc 2 2 hc/LkT
(
)
e
L
kT 2

Long bar

A long rectangular bar at temperature T exerts a tension, t(, T ), when it is


extended an amount beyond its natural length.
(a) What is the thermodynamic relation between dS, T, dE, t(, T ) and d,
where S is the entropy and E is the internal energy of the bar?
(b) What is the relationship between (S/)T and (t/T ) for a quasistatic
process?
(c) Let t(, T ) = b(1 T ), where b and are constants. What is the change
in internal energy when the bar is stretched from = 0 to = 0 at
constant temperature?
(d) What is the sign and magnitude of the heat, Q, necessary to maintain the
bar at constant temperature during this process?
Solution
(a) When the bar is extended the work it does is
dW = td
So
dE = dQ dW = T dS + td for reversible process.
(b) For quasistatic (reversible) process, define
F = E TS
then
dF = SdT + td
t
S
)|T = (
)|
(

T
(c) S can be expressed by S = S(T, ).
dE = T dS + td = T [(

S
S
) dT + (
)T d] + td
T

When dT = 0,
dE = (T (

S
t
)T + t)d = (T (
) + t)d

dE = [T (b) + b(1 T )]d = bd


Z 0
b 2
E =
bd = 0
2
0
Problem 6 from UCLA 1991Fall.

(d) We only consider the heat transfer. Its positive if it is from outside to bar.
Z 0
td
Q = E W = E
0
0

Z
bd

td
0

0
0

Z
=

d =

bT
0

So the sign is positive in our convention.

1
bT 02
2

10

Bose statistics

Assume in a system of volume V at temperature T there are excitations


which obey Bose statistics with = bp3/2 , where is the energy and p is the
momentum. The number of excitations is not conserved.
(a) Derive an expression for the temperature dependence of the heat capacity,
C, of the system. Express your answer in terms of Plancks constant and
T , leaving any integral in dimensionless form.
(b) What would be the temperature dependence of C, if the system were in a
space of arbitrary dimension D?
Solution
(a) Since the number of excitations is not conserved (like photon),
N () =

()

e kT 1

where N ()d is the average number of excitations of energy between and


+ d, and () is the density of states.
()d

V
d3~k
(2)3
V
d3 p
~
(2)3
h3
V
4p2 dp
(2)3
h3
V
4
d(p3 )
(2)3
h3 3
4 2
V
d( )
b
(2)3
h3 3
V
4 2
d
(2)3
h3 3 b 2

=
=
=
=
=
=

(19)
(20)
(21)
(22)
(23)
(24)

The total energy is

Z
U

N ()d

(25)

Z
=

4 22
V
d
(2)3 h
3 3 b2

e kT 1
Z
V (kT )3 x2 dx
3
3 2
h b2 0 ex 1

(26)

=
=
C
Problem 7 from UCLA 1991Fall.

2V (kT )3
3 2
h3 b 2
U
2V k3 T 2
= 2 3 2
T

h b

(27)
(28)
(29)

(b) If in arbitrary space dimension D,


()d

2D

dD~k d(pD ) d 3
Z
Z
2D
2D
()
d 3
d
(kT ) 3 +1

e kT 1
e kT 1
2D
U
T 3
T

(30)
(31)
(32)

Bose statistics

Assume in a system of volume V at temperature T there are excitations


which obey Bose statistics with = bp3/2 , where is the energy and p is the
momentum. The number of excitations is not conserved.
(a) Derive an expression for the temperature dependence of the heat capacity,
C, of the system. Express your answer in terms of Plancks constant and
T , leaving any integral in dimensionless form.
(b) What would be the temperature dependence of C, if the system were in a
space of arbitrary dimension D?
Solution
(a) Since the number of excitations is not conserved (like photon),
N () =

()

e kT 1

where N ()d is the average number of excitations of energy between and


+ d, and () is the density of states.
()d

V
d3~k
(2)3
V
d3 p
~
(2)3
h3
V
4p2 dp
(2)3
h3
V
4
d(p3 )
(2)3
h3 3
4 2
V
d( )
b
(2)3
h3 3
V
4 2
d
(2)3
h3 3 b 2

=
=
=
=
=
=

(1)
(2)
(3)
(4)
(5)
(6)

The total energy is

Z
U

N ()d

(7)

Z
=

4 22
V
d
(2)3 h
3 3 b2

e kT 1
Z
V (kT )3 x2 dx
3
3 2
h b2 0 ex 1

(8)

=
=
C
Problem 7 from UCLA 1991Fall.

2V (kT )3
3 2
h3 b 2
U
2V k3 T 2
= 2 3 2
T

h b

(9)
(10)
(11)

(b) If in arbitrary space dimension D,


()d

2D

dD~k d(pD ) d 3
Z
Z
2D
2D
()
d 3
d
(kT ) 3 +1

e kT 1
e kT 1
2D
U
T 3
T

(12)
(13)
(14)

Thermodynamic engine

A peculiar thermodynamic engine operates according to the following diagram:


All lines in the above thermodynamic path are straight lines. AB is horizontal

and BC is vertical.
(a) Sketch what this diagram would look like in the p V plane.
(b) Suppose the gas used in the engine is an ideal gas with specific heat
CV =const per mole. Calculate the equation for the segment AC in the
p V plane. [This means obtain a function p(V ) which describes the
segment AC. Your answer may involve constants like SA , SC , VA , VC and
n (number of moles).]
Solution
(a) See the diagram.

(b)
dE = T dS P dV
dE = nCV dT
nRT
P =
V
Problem 10 from UCLA 1991Spring.

nRT
dV
V
nCV
nR
dS =
dT +
dV
T
V
V
T
+ nR ln
S(V, T ) = S(V0 , T0 ) + nCV ln
T0
V0
nCV dT = T dS

along AC:
VC SA VA SC
SA SC

V
VC VA
VC VA
SA SC
T
V
VC SA VA SC

V = SA + nCV ln
+ nR ln
VC VA
VC VA
TA
VA
This can be solved for T = T (V ),
S=

nCV ln

T
VC SA VA SC
SA SC
V
=

V SA nR ln
TA
VC VA
VC VA
VA

1 VC SA VA SC
SA SC
V
(

V SA nR ln
)]
nCV
VC VA
VC VA
VA
1 VC SA VA SC
nRTA VA CR
SA SC
nRT
=
(
) V exp[
(

V SA )]
P =
V
V
V
nCV
VC VA
VC VA
T = TA exp[

Free expansion

Consider N atoms of an ideal gas in thermal equilibrium at temperature T


confined by a barrier to a fraction f of the volume of an adiabatic enclosure.
The barrier ruptures at some instant, and the gas expands to fill the whole
volume, and eventually reaches a new equilibrium state.
(a) What is the new temperature of the gas?
(b) What is the change in entropy of the gas?
Solution
(a) For ideal gas, when the barrier ruptures, the expansion is free, i.e. W = 0,
U = Q W = 0
U
U
)T,N dV + (
)V,N dT
V
T
U
( V
)T,N
) P)
(T ( P
dT
T V

|U = U
=
dV
( T )V,N
( U
)
T V,N
dU = 0 = (

P
Nk
P
N kT

=
T(
)V P = 0
V
T
V
T
dT
(
)U = 0
dV
So the temperature does not change from the initial temperature.
P =

(b)
dU = 0 = T dS P dV

S =
fV

dS
P
=
dV
T

P
dS = dV
T
Z
P
1 V N kT
1
dV =
dV = N k ln > 0
T
T fV V
f

Problem 6 from UCLA 1992Spring.

Ideal gas with a surface

Consider an ideal gas of classical monatomic particles in the presence of a


surface. The surface consists of N adsorption sites, each of which can accommodate at most one atom. The absorbed atoms are bound to the sites with
energy W , where W > 0. The gas is at density n and temperature T and is
in equilibrium with the surface. What fraction of adsorption sites is occupied?
Solution
The grand partition function of the gas is
X ( )
l
ln Z =
e
l

V
= 3
h
=e

ep

/2m 3

d pe

V3

where
=(

h2
)1/2
2mkT

is the thermal wavelength.


1
ln Z

V
= e 3

= (T )3 n

hN i =

e
For the sites,
Z=

N
X

l
e(+W )l CN

l=0

= (1 + e(+W ) )N
ln Z = N ln(1 + e eW )
hN i =

1
ln Z

N e eW
1 + e eW
1
= N ( W
)
e
e
+1

Problem 13 from UCLA 1992Fall.

Einstein model

The Einstein model of the lattice vibrations of a solid consisting of N atoms


represents the solid by 3N identical one-dimensional quantum harmonic oscillators, each with frequency 0 . Answer the following questions related to this
model:
(a) Find the mean energy of the system as a function of the temperature, T ,
of the solid.
(b) Find the heat capacity of the system, and evaluate it in the limit kB T 
h0 . Discuss this result in terms of the equipartition theorem.

(c) Find the general expression relating the pressure of a system to its Helmholtz
free energy.
(d) To model anharmonic effects in the solid, one assumes that the frequency
0 is a function of the volume, V . Find the pressure in the Einstein solid
as a function of (0 /V )T .
Solution
n =
h0 (n +

1
), n = 0, 1, ...
2

The partition function is


Z=[

eh0 (n+ 2 ) ]3N = lim [e

h0
2

n=0

1 xM 3N
)]
1x

where x = eh0 . Thus


ln Z = 3N [
E=

h0
ln(1 eh0 )]
2

ln Z
h0

h0

= 3N [
+ h
]

2
e 0 1

h0 2 eh0

dE
= 3N kB (
)
dT
kB T (eh0 1)2
At high temperatures
h0  1, thus
C=

C 3N kB

(
h0 )2
= 3N kB
(
h0 )2

From equipartition, for each 1D oscillator,


=

1
1
kB + kB T = kB T
2
2

thus =3N kB T and C = 3N kB .


F = E TS
dQ = T dS = dE + pdV
Problem 6 from UCLA 1993Spring.

thus
dF = d(E T S) = SdT pdV
F
(
)T = P
V
F = kT ln Z,
so
P = 3N
h(

1
1
0
)T [ + h
]
V
2
e 0 1

Bosonic excitations

The bosonic low-energy excitations of a two-dimensional system of dimensions


L L are described by the wave equation

2u
+ C4 u = 0
t2

where 4 = (2 )2 . (The scalar u(r, t) might represent height fluctuations normal to the two-dimensional plane.)
(a) Solve for the dispersion relation (k).
(b) Compute the density of states g().
(c) Compute, to within a numerical constant, the low-temperature specific
heat C(T ). (You may assume that kB T is much greater than the spacing
between neighboring quantized energy levels.)
Solution
(a) The wave equation is

2u
+ C2 2 u = 0
t2

so substituting u(r, t) = u0 ei(k~rt) gives 2 + Ck4 = 0, i.e.


r
C 2
k
=

where k = |~k|.
(b) The density of states is given by
L2
L2
2kdk =
2
4
4

g()d =

d
C

which gives
g() =

L2
4

,
C

a constant.
(c) The mean energy is
Z
E(T ) =

dg()
h[n() +
0

1
]
2

where n() = (eh /kB T 1)1 is the Bose occupancy factor. Note that E(T )
is infinite! This is because of the zero point energy. (If we had imposed an
ultraviolet (short wavelength) cutoff on the density of states, as in the Debye
model, this wouldnt have happened.) However, the infinite zero point energy is
Problem 10 from UCLA 1993Spring.

temperature independent and doesnt affect the specific heat. The T -dependent
part to E(T ) is
r Z
L2
1
E(T ) E0 =
d
h h /k T
B
4 C 0
e
1
r
Z
x
L2 (kB T )2
dx x
=
4 C
h

e 1
0
r
2
2
2
L
(kB T )
=
6 4 C
h

where the integral gives the numerical constants 2 /6. The specific heat is then
linear in T :
r
2
E
2 L2 2kB
T
C(T ) =
=
T
6 4 C
h

Refrigerator in air conditioned room

Consider an air conditioned room with a refrigerator inside. The temperature


in the refrigerator is maintained at T1 , in the room the temperature is maintained at T2 , and the outside temperature is T3 . The temperatures are related
by T1 < T2 < T3 . All heat produced by the refrigerator goes into the room and
should be pumped out by the air conditioning unit to maintain the temperature
T2 .
(a) A massive object is brought from outside and placed inside the refrigerator.
The object has heat capacity C. Calculate the total work W done by both
the refrigerator and the air conditioner while cooling the object to T1 .
Assume that both the refrigerator and the air conditioning unit are ideal
heat machines.
(b) Now consider another experiment. The same object is brought from the
outside into the room. First it naturally cools to room temperature T2 .
Next it is placed in the refrigerator and cooled to T1 . Will this require the
same amount of work done by both the refrigerator and the air conditioner
as in part (a)? If not, calculate the total work done. Which amount of
work is smaller?
Solution
First we recall that an ideal heat machine is based on the Carnot cycle. In the case of
refrigeration the work must be done to move heat from cold to hot objects. Consider
a cycle between a cold refrigerator interior (system 1) and hot room (system 2). The
total entropy of the entire system (1+2) does not change so S1 + S2 = 0. The cycle
involves an isothermal expansion at temperature T1 and an isothermal compression at
the temperature T2 . This process involves a heat transfer Q1 = T1 S1 is taken from
refrigerator (since Q1 < 0) and Q2 = T2 S2 > 0 is placed in the room. The amount
of work required as follows from the energy conservation is


T2
1 .
(1)
W = Q1 + Q2 = Q1
T1
We thus find that cooling an object with heat capacity C from T2 to T1 requires
W =C

(T2 T1 )2
.
T1

(2)

(a) When a massive object with temperature T3 is placed in the refrigerator cooling
it to T1 requires Q1 = C(T3 T1 ). The minus sign indicates that the heat is to
be removed from the system 1. We already know that this removal reduces the
entropy in system 1 and raises the entropy in the room by the same amount. This
means that heat brought by the refrigerator into the room is Q2 = Q1 T2 /T1 .
Finally, this heat is to be pumped outside (we move Q2 ) which gives to the
outside world
T3
T3
Q3 = Q2
= Q1
(3)
T2
T1
http://www.phys.ufl.edu/academics/downloads/jan11-partb.pdf

The amount of work to be done on the system is


W = (Q1 + Q3 ) = C

(T3 T1 )2
.
T1

(4)

The work done by the refrigerator is



WR = C(T3 T1 )


T2
1 ,
T1

and the work done by the airconditioning








T3
T2 T3
T2 T3
WAC = Q2
1 = Q1
1 = C(T3 T1 )
1 .
T2
T1 T2
T1 T2

(5)

(6)

This result shows that the two ideal heat machines working sequentially are
equivalent to a single ideal machine. Indeed, here we have a problem of a
heat transfer from one system at the temperature T1 to another system at the
temperature T3 . The minimum amount of work needed to to that is achieved in
the Carnot cycle.
(b) The situation in the second experiment is different. While the same amount of
heat |Q1 | = C(T3 T1 ) is taken away from the massive object this heat is taken
away at different temperatures. At intermediate temperatures the efficiency of
the Carnot cycle is higher and heat transfer takes less work. In the first step
the work done by the AC unit is


T3
W (1) = C(T3 T2 )
1 .
(7)
T2
In the second step simply amounts to part (a) where one has to remove C(T2 T1 )
amount of heat from refrigerator to outside.


T3
W (2) = C(T2 T1 )
1 .
(8)
T1
Clearly the total amount of work is smaller because. First the C(T3 T2 )
amount of heat is removed with inverse coefficient of performance (inverse of
efficiency) T3 /T2 1 then a second amount of heat C(T2 T1 ) is removed with
inverse coefficient of performance (T3 /T1 1) Since the first part is more efficient
(meaning less work is needed) than that of (T3 /T1 1) in part (a) the second
method will require less work.
The minimum amount of work needed to cool an object with heat capacity C
from temperature T3 to some lower temperature T1 is
 
T3
C(T3 T1 ).
(9)
W = CT3 ln
T1

Semiconductor

A simple model of a semiconductor is to assume that it has M donor electron


levels, all at a negative energy, , and a free particle electron conduction
band whose minimum energy is zero. Assume that at zero temperature the
conduction band is empty, and that all the donor levels are filled, with one
electron per donor. At finite temperature T , a number N of the electrons are
thermally excited from the donor states to the conduction band, leaving behind
N holes in the donor levels. Assume N << M throughout and ignore electronelectron interactions. Let the volume of the semiconductor be V .
(a) Determine the energy, entropy, and (Helmholtz) free energy of the electrons in the conduction band. In doing so, assume Maxwell-Boltzmann statistics and express the entropy in terms of N and the thermal wavelength,
th = (2h2 /mkB T )1/2 , where m is the electron mass and kB is Boltzmanns
constant. In terms of t h, what is the condition that the conduction electrons
obey Maxwell-Boltzmann statistics?
(b) Determine the total energy, entropy, and free energy of the M N electrons and N holes in the donor levels. Assume N >> 1 and use Stirlings
approximation, lnN ! ' N lnN N .
(c) Determine the equilibrium number of electrons excited to the conduction
band as a function of M , T , and , including the free energy of the electrons in
the donor levels, but neglecting the entropy of the electrons in the conduction
band.
(d) Determine the equilibrium number of electrons excited to the conduction
band as a function of M , T , and , including the free energy of the electrons in
the conduction band, but neglecting the entropy of the electrons in the donor
band.
(e) Determine the equilibrium number of electrons excited to the conduction
band as a function of M , T , and , taking into account all the terms in the free
energy of the electrons in both the donor levels and the conduction band.
Solution
(a) We first found the partition function Q1
Z 3 3
p2
d rd p 2m
e
Q1 =
3
h
0
The integral of d3 r was given as volume V . Performing the integral results in the
following
2mkB T 3/2
Q1 = V (
)
h2
2

h
It was given that th = ( 2mk
) 2 which gives the results
BT

Q1 =
Problem G1P3 from 2008Comps.

V
3

We know that the particles are indistinguishable, especially at higher temperatures


therefore
1 V N
( )
QN =
N ! 3
We can use this result to find the free energy (Helmholtz energy) which is defined as
A = kB T lnQN
V
A = N kB T [ln(
) + 1]
(10)
N 3
The energy is then defined as
E

d
lnQN
d
d
V
ln 3
d
d
2mkB T 32
lnV (
)
d
h2

Which results in the energy for the electrons in the conduction band as
E=

3
N kB T
2

(11)

Having already found the Helmholtz free energy and the energy and knowing that
E = A + T S, we can solve for the entropy of the electrons in the conduction band.
S=

V
3
N kB + N kB [ln(
) + 1]
2
N 3

(12)

The condition that the conduction electrons obey Maxwell-Boltzmann statistics is


based on the number of degrees of freedom.  V 1/3
(b) We first start by finding the partition function with the choose function to normalize the equation
QN = (M chooseM N )e(M N )
Once again we use A = kB T lnQN to find the Helmholtz energy.


M!
A = kB T ln
e(M N )
(M N )!N !
M!
A = kB T (ln
+ lne(M N ) )
(M N )!N !
A = kB T (ln(M !) ln(M N )! ln(N !) + ((M N )))

Expand using Stirlings approximation lnN ! ' N lnN N


A

kB T (M lnM M (M N )ln(M N ) + (M N ) N lnN + N + (M N ))

kB T (M lnM M M ln(M N ) + N ln(M N ) + M N N lnN + N + (M N ))

kB T (M lnM M ln(M N ) + N ln(M N ) N lnN + (M N ))


M
M N
kB T (M ln
+ N ln
+ (M N ))
M N
N

N
is zero since M  N and N ln MN
can be changed to N ln M
N ln1,
The ln MM
N
N
which the ln1 = 0. Therefore, the free energy is

A = kB T (N ln

M
+ (M N ))
N

(13)

We define the energy as


E

d
lnQN
d
d
M
[ln
+ (M N )]
d
N

The resulting energy for the donor electrons is


E = (N M )

(14)

Once again, to find the entropy we use equations (4) and (5) and E = A + T S
S = N kB ln

N
M

(15)

(c) Using the Euler relations, dE = T dS P dV + dN and dA = SdT P dV + dN ,


we find that the chemical potential for the conducting plane (neglecting the entropy
dE
of the electrons in the conduction band) is C = dN
. Using equation (2), we find
C

d 3
N kB T
dN 2
3
kB T
2

The chemical potential for the donor plane taking into consideration the free energy of
T d(MdSN ) . Using the results
the electrons in the donor level results in D = d(MdE
N )
from equations (5) and (6) we solve for the chemical potential of the donor plane.

d
d
N
(N M ) + T
N kB ln( )
d(M N )
d(M N )
M
M
+ kB T ln( ) kB T
N

We set the chemical potentials equal and solve for N, the equilibrium number of electrons excited to the conduction band.
D
M
+ kB T ln( ) kB T
N
M
ln( ) 1
N
M
N

=
=
=
=

C
3
kB T
2

3
+
kB T
2
5

e 2 +

The number of electrons excited to the conduction band are as follows


5

N = M e( 2 +)

(16)

(d) Once again we use the Euler relations and equations (2) and (3) to find the chemical
potentials of the conducting band.
C

dE
dS
T
dN
dN
V
kB T ln
N 3

We use the Euler relations and equation () to find the chemical potentials of the
conducting band.
D

dE
d(M N )
d
(N M )
d(M N )

We set the chemical potentials equal and solve for N, the equilibrium number of electrons excited to the conduction band.
D

kB T ln

V
N 3

The number of electrons excited to the conduction band are as follows


N=

V
e
3

(17)

(e) To find the equilibrium number of electrons we set the chemical potential of the
conduction bands from part C and part D equal and solve for N.
D (partc)
M
+ kB T ln( ) kB T
N

C(partd)

kB T ln

V
N 3

The equilibrium number of electrons taking into account all the terms in the free
energy of the electrons in both the donor and the conduction bands are
r
M V 21 (1+)
N=
e
3
Replacing ,
r
N=

MV (

2mkB T 3/2 21 (1+)


) e
h2

(18)

Ice in water

100g of ice at 20C is put into 100g of water at 5C. Assuming no heat flow
to or from external sources, Describe exactly what happens and give reasons for
your response.
Cice = 2090J / kg o C

Cwater = 4186J / kg o C

Lf usion = 3.33 10 J / kg

Lvaporization = 2.26x10 J / kg

(19)
(20)

Solution
We know that at equilibrium the temperature for the two objects will be the same.
Based on looking at the specific and latent heats, we have reason to suspect the
equilibrium point will be somewhere along the Tf = 0 line. So we guessed that the
equilibrium was 0 and that some of the water would freeze. So well try it and see
whether our answer satisfies the conditions.
So heres what happens: Twater = 5C, mass 100g, temperature of water reduces to
0o C and the temperature of the ice goes to some unknown temperature Ti .
Heat gained by ice during this time = Heat lost by water
m Ci (Ti + 20) = m Cw 5
Solving for Ti we have the intermediate temperature in the process at which the water
will begin to freeze in the cup. It turns out to be,
Ti 10o C.
Next, some percentage () of the water is turned into ice.
Heat lost freezing of the water = heat gained by ice going from 10o C to 0o C
(21)
m l f = m Cw 5

(22)

Solving for we obtain:


= 6.25%

(23)

At this point the two systems are at equilibrium with a temperature of 0o C so the
mixture will remain as both solid and liquid.

Problem G1P14 from 2008Comps.

Otto cycle

An Otto cycle engine burns gasoline and goes through a reversible process
consisting of two adiabatic, and two isovolumetric parts in the PV diagram
shown in the problem. (a) Show that the efficiency of the engine can be written
c
as e = 1 TTab T
Td . (b) Show that the efficiency can also be written as
V1 1
1
= 1 1 .
V2
r

e=1
where r =

V2
V1

Cp
Cv .

Solution
(a)
e=

dW
dQh

(24)

But by energy conservation the work done is the heat taken from the hot reservoir and
dumped to the cold reservoir. Therefore,
(dQh + dQc )
dQh
(dTh dS + dTc dS)
=
dQh dS

e=

(25)
(26)

giving us the classic result:


e=1

dTc
.
dTh

(27)

e=1

Tb Tc
.
Ta Td

(28)

or,

(b) For adiabatic processes we know that P V = constant, therefore,


P1 V1 = P2 V2
P1
V = V2
P2 1
but from the ideal gas law,

P1
P2

T1
T2

T1
T2

V1
V2

V1
V2

1

1

(29)
(30)

, thus we have,

V1 = V2

(31)

T1 V11 = T2 V21 .

(32)

Problem G1P14-2 from 2008Comps.

But since this has to be true for the two adiabatic processes, we have two equations:
Ta V11 = Tb V21

Td V11 = Tc V21

subtracting these two equations from each other we are left with,
(Ta Td ) V11 = (Tb Tc ) V21
therefore,
Ta Td
=
Tb Tc

V1
V2

1

Thus we may plug this into our previous result (equation 10) to obtain, e = 1

V1 1
V2

Rotational gas

Consider the rotational motion of a dilute gas of N diatomic molecules, each


with moment of inertia I. In this problem you are asked to consider three cases:
a quantum system of diatomic molecules in which the two atoms are distinguishable (parts A-C); a quantum system of diatomic molecules in which the
two atoms may be considered as indistinguishable spinless bosons (part D), and
a classical system of rotors (part E). In this problem we will consider rotational
motion only. You may use the fact that a quantum rotor has energy levels
2
EJ = h2I J(J + 1).
In parts A-C, use quantum statistical mechanics for the case where the two
atoms in each molecule are distinguishable.
(a) Give an expression for the partition function Z as a function of temperature
T. You do not need to evaluate the expressions in a closed form. Be sure to
properly include all degeneracy factors in your expression.
(b) When the temperature T is low but non-zero, derive an asymptotic expression for the rotational specific heat per molecule C as a function of temperature.
(c) In the limit of high temperature, the expression for Z in part (a) can be approximated as an integral over a continuous variable. By evaluating the integral,
give the limiting value of the specific heat per molecule C.
In part (d) consider the case where the atoms in each molecule can be regarded
as indistinguishable spinless Bosons.
(d) Give the expression for the partition function Z in this case which replaces
the one in part (a). Explain your reasoning. You do not need to evaluate the
expression.
In part (e), consider the relation to classical statistical mechanics.
(e) The expression for the specific heat using quantum statistical mechanics
should approach the classical result in the high temperature limit. Show that
your result of part (c) does approach the correct classical limit in which the
specific heat is determined by the number of degrees of freedom for the rotational
motion.
Solution
(a) The particles are distinguishable so the partition function is given by
Z = (Z1 )

!N
EJ /kB T

gJ e

J=0

where gJ is the degeneracy factor. The quantum rotor energy levels have a degeneracy
of 2J + 1. Hence, we have
!N


h2 J(J + 1)
.
(33)
Z=
(2J + 1) exp
2IkB T
J=0
Problem G2P8 from 2008Comps.

(b) For kB T  h2I , we should keep only the the first two terms (J=0 and J=1) in
the partition function to get the leading order temperature dependence of the specific
heat. This gives us

N
2
Z = 1 + 3eh /I
(34)
for the partition function. First, we calculate the internal energy
2

2
ln(Z)

3N
h2 eh /I
3N
h2 h2 /I
U =
= N
ln 1 + 3eh /I =

e
.
2

I 3eh /I + 1
I
Notice that it is sufficient to keep only the leading order temperature dependence of
the energy above. Lastly, we find the specific heat per molecule at low temperature
 2 2
2
1 dU
h

1
dU
C=
=
=
3k
eh /IkB T .
(35)
B
2
N dT
N kB T d
IkB T
(c) For kB T 

h
2
,
2I

we can write


Z

h2 J(J + 1)
.
Z1
dJ(2J + 1) exp
2IkB T
0

(36)

Let
dx
= 2J + 1.
dJ
Substituting these last two expressions into equation (4) we get


Z

h2 x
2IkB T
Z1
dx exp
=
.
2Ik
T
h2

B
0
x = J(J + 1)

(37)

Calculating the energy we obtain


U
1 Z1
1
=
= = kB T.
N
Z1

For the limiting value of the specific heat per molecule, we have then
1 dU
= kB .
(38)
N dT
(d) The total wave function for bosons must be symmetric with respect to interchange
of identical particles. Recall that we are considering rotational motion only in this
problem. Hence, the eigenfunctions of our Hamiltonian are the spherical harmonics.
We use the following property of the spherical harmonics
C=

P YJm (, ) = (1)J YJm (, ),


where P is the parity operator. This means that we should keep only the J even states
so that we have a symmetric wave function under particle exchange. Therefore,
!N
!N


X
X

h2 J(J + 1)
EJ /kB T
=
(2J + 1) exp
.
(39)
Z=
gJ e
2IkB T
J=0,2,4,...
J=0,2,4,...
(e) Here we use the equipartition theorem. The rigid rotor Hamiltonian has two
quadratic degrees of freedom f. Using classical theory we obtain for the specific heat
per molecule
f
2
C = kB = kB = kB .
(40)
2
2
This agrees with the limiting value of the specific heat per molecule that we obtained
in part (c).

Diatomic molecules

Consider a classical gas of N diatomic molecules in thermal equilibrium with


a heat bath at temperature T. The molecules move freely inside a cubic box of
volume V. Each molecule consists of two atoms of mass m1 and m2 respectively.
The two atoms are permanently bound together inside the molecule by intramolecular forces with potential
 r 
U (
r) = U0
a
where r is the relative separation of the two atoms, U0 is a positive constant
with units of energy, a is a characteristic length, and is a positive constant.
The molecules do not interact with each other.
j ,
Let j = 1, 2...N label the molecules, and for the j th molecule denote by R
Pj , rj , and pj the coordinates of the center of mass, the momentum of the center of mass, the relative separation of its component atoms and their relative
momentum, The classical Hamiltonian is
#
"
N
X
p~2j
P~j2
+
+ U (rj )
E=
2M
2
j=1
where M = 21 (m1 + m2 ) and = m1 m2 /(m1 + m2 ) .
(a) Show that the classical partition function ZN is given by
ZN

VN
=A
N!

kB T
2h2

3N Z

2 U (
r )/kB T

N

dr 4
r e
0

And express the constant A in terms of M and .


(b) In the thermodynamic limit N 1, N/V fixed, calculate the average energy
in therms of N, kB T and . [Hint: the change of variable t U (r)/kB T
may be useful. You should not need to evaluate any integrals for this
part.]
(c) Calculate the heat capacity at constant volume CV .
(d) For the case = 2, explain how the answer to (c) could have been obtained
without detailed calculation.
Solution
(a)The 1-particle partition function is determined by
Z
Z1 =
Problem G4P5 from 2008Comps.

d3 P d3 p d3 R d3 r E
e
h6

(41)

Notice that the Hamiltonian is independent of R. Thus, integrating over the


relative coordinate results in a factor of volume V. This leaves
Z 3
Z 3
Z
~2
p
~2
P
j
j d3 p
d P 2M
d p 2
Z1 = V
e
e
d3 r eU (r)
(42)
h3
h3
h3
The first two integrals are common in statistical mechanics. They can be preformed easily by integrating in spherical coordinates. This results in

Z1 = V

2M kB T
h2

3 
2

3 Z
2

2kB T
h2

dr 4
r2 eU (r)

(43)

ZN

The partition function for N particles is just N1! because the particles are noninteracting and indistinguishable. Thus,

3N Z
N
V N kB T
2 U (
r )/kB T
ZN = A
d
r

4
r
e
(44)
N ! 2
h2
0
Where
A (M )3N/2
(b) Let

 r 

t U0


r=a
1 a
dr =
U0

t
U0

(45)

(46)

1/
(47)

t
U0

 1

dt

(48)

Now, the N-particle partition function becomes


ZN = (M )3N/2 V N

kB T
2
h2

3N

a3N
N

kB T
U0

3N/ Z

dt t

et

(49)
This integral is just a finite constant. The average energy is given by
= kB T 2 d ln(ZN )
E
dT


1
+

=3
E
N kB T

(50)
(51)

(c) Finding the heat capacity is trivial

E
T

1+


CV =

CV = 3

(52)
V

N kB

(53)

(d) When = 2, the Hamiltonian only depends on quadratic terms. This means that
the equipartition theorem is valid. In the Hamiltonian there are nine independent quadratic degrees of freedom - three from the center of mass momentum,
three from the relative momentum, and three from the potential energy expression. The equipartition theorem states that
E=

f
N kb T
2

(54)

where f is the number of quadratic degrees of freedom. This is 9 for our case:
E=

9
N kb T
2

(55)

meaning that the specific heat is


9
N kb
2
These expressions agree exactly with equations (52) and (54).
CV =

(56)

Classical ideal gas

For a classical ideal gas the number of molecules having component momenta
between px and px + dpx , py and py + dpy , and pz and pz + dpz is

fN =

 P 2
p
N
exp
i i d3 p
2mkT
(2mkT )3/2

(a) Show that the total number of particles is N.


(b) Find the number of particles having speeds between v and v + dv.
(c) If the mean free path ` is independent of the speed v, the mean time between
collisions for a particle of speed v is `/v. Calculate the mean time between
collisions for particles of all speeds.
Does this result = `/(v 2 )1/2 ?
Please Note:

x2

dx =

Solution
(a) Denote the total number of particles by M. We know that
Z
M = f (p)d3 p,

(57)

where f(p) is the number of particles with momentum between p and p + dp.


ZZZ
(p2x + p2y + p2z )
N
exp
dpx dpy dpz .
(58)
M=
2mkT
(2mkT )3/2
Doing the integration, we get
M=


3
N
2mkT
= N.
(2mkT )3/2

(59)

(b) We know p = mv. Replacing the momentum in given distribution function by


this equation and using the fact that dp = mdv, we get


 m 3/2
m(vx2 + vy2 + vz2 )
F (v) = N
exp
dvx dvy dvz .
(60)
2kT
2kT
In spherical coordinates we obtain
F (v) = 4N
Problem G4P16 from 2008Comps.



 m 3/2
mv 2
v 2 exp
dv.
2kT
2kT

(61)

(c) To find mean free time we use


Z

`
F (v)dv.
vN


 m 3/2Z
mv 2
= 4`
vdv exp
.
2kT
2kT
=

(62)

(63)

This integral returns


r
m
= 2`
.
2kT
Finally, we must calculate `/(v 2 )1/2 .


Z
 m 3/2Z
mv 2
3kT
4
2 F (v)
2
dv = 4
v dv exp
.
v = v
=
N
2kT
2kT
m

(64)

(65)

Hence we have,
r
`
m
=`
.
2
1/2
3kT
(v )
This is not the same result that we obtained for the mean time .

(66)

Van der Waals gas

The Van der Waals equation of state for one mole of a nonideal gas is (P +
)(V b) = RT . Show that the entropy for this gas is S = Cv lnT + Rln(V
b) + const. Use this result that, for an adiabatic change in a Van der Waals gas,
R
T (V b) Cv = constant.
a
V

Solution
We start off with the partial equation for entropy.
dS =
We know that E = T S and Cv =

E
.
T

dS =
Substitute

Cv
T

S
S
dV +
dT
V
T
Therefore,

Cv
S
dV +
dT
V
T

dT for Cv ln(lnT ).
dS =

Using the relation

S
V

P
T

S
dV + Cv d(lnT )
V

we can substitute into the equation.


dS =

P
dV + Cv d(lnT )
T

We can use the initial equation to easily solve for


dS =
R
dV
V b

T
P

V b
,
R

Therefore,

R
dV + Cv d(lnT )
V b

can be expressed as Rln(V b).


dS = dln(V b) + Cv d(lnT )

We integrate through to find our final answer, which includes an integration constant.
S = Cv lnT + Rln(V b) + const
Now we must show that for an adiabatic process (S = constant), S = Cv lnT +Rln(V
R

b) + const can be expressed as T (V b) Cv = constant. To do so, we know that any


exponent in logarithmic function can be brought down in front of the natural log.
Thus we can reverse this process to produce.
constant = lnT Cv + ln(V b)R + const
Using the logarithmic multiplication rules,
Constant = ln(T Cv (V b)R )
Problem G5P14 from 2008Comps.

Now we place both sides to the exponential to cancel out the natural log.
constant = T Cv (V b)R
Since the specific heat is remaining constant, we can multiply the exponents by
resulting in the final answer of
R

constant = T (V b) Cv

1
Cv

1d polymer

Let us consider a polymer consisting of N(a positive even integer) monomers


connected end-to-end and lying along the x-axis. The monomers may be regarded as small arrows of lengths l without thickness and lie along the x-axis
with no preferred direction.One end of the polymer is fixed at the origin O.

(a) Find the number N+ of monomers pointing in the positive direction long
the x-axis, and the number N of monomeres pointing in the negative
direction along the x-axis, when the free end x-coordinate is X = 2nl,
where N/2 n N/2.
(b) Write down the total number N (X) of allowed conformations when the
free end x-coordinate is X = 2nl as in (a).
(c) Let SN (X) be the conformational entropy of our polymer whose end xcoordinate is X. Compute the entropy difference SN (X) SN (0) to order
X 2 , assuming that | n |<< N .
(d) Find the required average force in pN (picoN ewton = 1012 N to make the
end-to-end distance of the polymer be R = 200
A. Assume that N = 4
104 , the monomer size l is 1
A(= 11010 m), and the ambient temperature
is 300K. Assume that the only contribution to the forces comes from
the conformational entropy of the polymer for which you may use the
functional form obtained in (c
A. Do not hesitate to use thermodynamic
considerations, if microscopic calculations can be avoided.
Solution
(a) We know that the total number of monomers is given by:
N = N+ + N

(67)

We want to find the N and N+ when X = 2nl. We have that:


X = l(N+ N )
Problem problem1 from 2008Comps.

(68)

Taking equation 67 and solving for N we get


N = N N+

(69)

Substituting in equation 68 and solving for N+


N+ =

N
X
+
2
2l

(70)

Substituting equation 70 in equation 69 we have that:


N =

X
N

2
2l

(71)

(b) For the total number of N (X) we have that N (X) =


N+ and N we have
N (X) =

N
2

X
2l

N!
 N
! 2

X
2l

N!
.
N+ !N !

Substituting

(72)


!

(c) We are going to call the entropy difference S. But first we want to get S(X),
for that we use the relation SN = KB ln N
Substituting equation 72 we have
"
SN (X) = KB ln

N
2

N!
 N
! 2
+X
2l

#

X
!
2l

Using the approximation M ! ' 2eM M M we have

2eN N N
i h
SN (X) = KB ln h
N
X
N
N
X
2e( 2 + 2l ) ( N + X )( 2 + X
)
2l
2e( 2 2l ) ( N2
2
2l

X
X (N
) 2 2l )
2l

Looking at this equation we can see that it can be written as


"
#
NN
SN (X) = KB ln
N
X
N
X
2( N2 + X
)( 2 + 2l ) ( N2 X
)( 2 2l )
2l
2l
(
SN (X) = KB

N ln N ln 2 ln

"

N
X
+
2
2l

( N + X ) #
2

"

2l

ln

N
X

2
2l

( N X ) #)
2

2l

Using some properties of logarithms

SN (X) = KB



 
 
 


N
X
N
X
N
X
N
X
ln N N ln 2
+
ln
+

ln

2
2l
2
2l
2
2l
2
2l

Factorizing N/2 in the logarithm




  
 
  


N
X
N
N
X
N
X
X
SN (X) = KB N ln N ln 2
+

ln
1+

ln
1
2
2l
2
Nl
2
2l
2
Nl


 
 
 


N
X
N
X
N
X
X

N ln N ln 2 N ln
ln 1 +

ln 1
2
2
2l
Nl
2
2l
Nl


 
 
 


N
X
N
X
X
X
SN (X) = KB ln 2 + N ln 2
+

ln 1 +

ln 1
2
2l
Nl
2
2l
Nl
SN (X) = KB

We know that the series for logarithmic function is given by


ln(1 + x) = x

x3
x4
x2
+

+ ....
2
3
4

1<x<1

Expanding the ln and keeping only to second order in X 2





 



N
X
X
X2
N
X
X
X2
SN (X) = KB ln 2 + N ln 2
+

2
2l
Nl
2N 2 l2
2
2l
Nl
2N 2 l2




X2
X2
SN (X) = KB ln 2 + N ln 2 +

2N l2
N l2



X2
SN (X) = KB ln 2 + N ln 2
2N l2
Therefore,
S =

KB X 2
2N l2

(d) The force is given by


F =

U
X

From thermodynamics we know that




U
T =
S N,V
Therefore


F = T

S
X

Substituting equation 73 we get


F =

T kB X
N l2

F = .207pN

N,E

(73)

10

Noninteracting atoms

A gas of N indistinguishable classical non-interacting atoms of mass m is held


in a neutron atom trap by a potential of the form V (r) = ar. The gas is in
thermal equilibrium at a temperature T.
(a) Find the partition function Z1 for a single atom trapped in this potential.
Express your answer in the form Z1 (T, a) = A T a . Find the exponents
of and , and the constant A.
(b) Find the entropy of the gas in terms of N , kB , and Z1 (T, a) = A T a .
(c) The gas can be cooled if the potential is lowered reversibly(by decreasing
a) while no heat is allowed to be exchanged with the surroundings. Find
T as a function of a and the initial values To and ao .
Solution
(a) The Hamiltonian is:
mH =

p2
+ ar
2m

(74)

Thus the partition function is,


d3 p d3 r H
e
h3
Z
Z
p2
1
= 3 4
e 2m p2 dp 4
e ar r2 dr
h
0
0
Z

Z1 =

letting x ar, and y


1
Z1 = 3 4
h

2m

2m

(76)

p, thus,

 3 Z
2

|0

Z
2
ex x2 dx
ey y 2 dy (a)3 4
0
{z
}
|
{z
}

= 4

(75)

3
8
(2 mkB T ) 2
3
h

kB T
a

(77)

=2

3
(78)

Therefore,
Z1 =
Thus, =

9
2

3
9
8
3
(2 mkB ) 2 kB
T 2 a3 .
h3

(79)

and = 3.

(b)

S=
=

A
T


(80)
V,N

(kB T lnZN )
T

Problem StatMech4 from 2008Comps.

(81)

but ZN =

1
ZN ,
N! 1

therefore,




1 N
Z1
T ln
T
N!

(T (N ln Z1 ln N !))
= kB
T
= kB

(82)
(83)

using Stirlings approximation ln N ! N ln N N ,


= N kB
but

Z1
T



1 Z1
(ln Z1 ln N + 1) + T
Z1 T

(84)

Z1 (where = 92 ), therefore,

S = N kB

(ln Z1 ln N + 1) + T

1
Z1
Z1 T

= N kB ((ln Z1 ln N + )

(85)
(86)
(87)

Therefore,

S = N kB


ln

Z1
N


+

11
2


.

(88)

(c) Since dQ = T dS = 0, and T is not equal to 0, then dS = 0. So,

So = S


 


 
Z1 (T, a)
Z1 (To , ao )
11
11
+
= N kB ln
+
N kB ln
N
2
N
2

(89)
(90)

Z1 (To , ao ) = Z1 (T, a)

(91)

A To a
= A T a
o
   
T
a
=
To
ao

(92)
(93)

Therefore,

T = To

a
ao

2
3

(94)

Molecule of four atoms


A molecule consists of four identical atoms at the corner of a tetrahedron.

(a) What is the number of (i) translational, (ii) rotational, and (iii) vibrational
degrees of freedom for this molecule?
(b) On the basis of the equipartition principle, what are the values of Cv and
= Cp /Cv for one mole of an ideal gas composed of these molecules?
(c) Assuming that the vibrations of the molecule are quantized, what is Cv for
temperatures such that kB T  hv ? Here v is the lowest frequency of
vibration of the molecule.
(d) At very low T the rotations of the molecule are also quantized, i.e. Erot =
(h2 /2I)l(l + 1), where l = 0, 1, 2... and I is the moment of inertia of the
molecule. What is the value of Cv at very low temperatures (kB T 
h2 /2I)? Assume the gas is still ideal at low T .

Solution
(a) We have four particles which have three positions each, as they are moving in
three dimensions. This gives a total of 12 degrees of freedom. Three of these
are translations of the center of mass, and since the molecule has no continuous
rotational symmetries (unlike a diatomic linear molecule, for example) then
there are also three rotational degrees of freedom. This leaves six of the original
twelve degrees of freedom which must be vibrational degrees of freedom. Note
that we may not have this many modes, but the sum of the number of modes
times the number of degrees of freedom within each mode will be six.
(b) The classical equipartition theorem states that for each independent quadratic
variable in the energy (Hamiltonian) we have a mean energy of kB T /2 when
that system is at a temperature T . Note this only holds when the level spacing
is small compared to kB T . The kinetic energy
T =

N
X
1 2
(vx + vy2 + vz2 )
2
i=1

has three such independent quadratic degrees of freedom for each particle, so it
has a mean of 3kB T /2 for each particle.
The same is true of the kinetic energy of rotation about the center of mass, since
it looks like
N
X

1
Trot =
I1 12 + I2 22 + I3 32
2
i=1
and so it also has a mean of 3kB T /2 for each particle.
The vibrational degrees of freedom enter the Hamiltonian with both a quadratic
kinetic energy term and a quadratic potential energy term. Each vibrational
Problem opathf99 from ProfDiag bank.

degree of freedom within a normal mode behaves like a one-dimensional oscillator


with energy
p2
1
H=
+ kx2 ,
2m
2
and so each vibrational degree of freedom contributes an average of 2 kB T /2 =
kB T to the energy.
Putting these together, we have (recall there is one mole of the gas)


3
3
U = NA
kB T + kB T + 6kB T = 9NA kB T = 9RT
2
2
so that
Cv = 9R.
For an ideal gas we have that Cp = Cv + nR so that for this mole of gas
Cp = Cv + R = 10R, so that
=

Cp
10
=
.
Cv
9

(c) At sufficiently low temperature the classical equipartition theorem no longer holds,
because kB T can become comparable to the level spacing of the oscillator (or
the rigid-rotator rotational spectrum). Since part d) implies that the rotational
motion has much smaller frequencies than the vibrational motion, we know that
for temperatures T so that kB T 
hv the vibrational oscillators are in their
ground state, and so the internal energy U associated with them does not depend
on T and we have


3
3
kB T + kB T ,
U = NA
2
2
and so Cv = 3R.
(d) Now even the rotational degrees of freedom are in their ground state and so the
energy associated with them does not depend on T , so all that is left are the
translational degrees of freedom and
3
U = NA kB T,
2
giving Cv = 3R/2.

Electron gas

Consider an ideal gas of N electrons that move freely in 2D, and are confined
to an area A.
(a) Estimate the temperature T0 below which quantum mechanical considerations are important.
(b) Calculate the Fermi energy assuming the system is at the temperature
T = 0K. Compare the Fermi energy to kB T0 , where T0 is calculated above.
(c) (c) What is the physical interpretation of the Fermi energy for a given
system?
Solution
(a) The area of a 2D system of N electrons is: A N (x)2. Therefore x
(1/)(A/N )1/2 . Using the uncertainty relation: xp
h, we can then write:
p
h/x.
To estimate the temperature below which quantum effects become important,
we will use:
E kB T0 (p)2 /2m
h2 /(2m(x)2 )
h2 N 2 /(2mA).

(1)

Therefore, as a crude approximation: T0


h2 N 2 /(2mAkB ).
(b) Calculate the 2D Fermi energy to compare to the energy scale in part (a):
ZkF
N = 2A
0

d2 k
=2A
(2)2

ZkF

2kdk
4A
=
(2)2
(2)2

ZkF
kdk =

A
kF 2
2

(2)

The factor of 2 is to account for the spin of the electron. The Fermi energy is
therefore:


h2 kF 2

h2 2N

h2 N
EF =
=
=
2m
2m
A
mA
The Fermi temperature T0 is then:
T0 =

kB

2N
h
mA

Note that this is close to the crude result estimated in part (a).
(c) The Fermi energy is a measure of the energy scale where quantum effects become
important for describing the properties of a system of Fermions. For most metals
at room temperature, for example, the Fermi energy is very large, and therefore
Fermi-Dirac statistics are needed to describe the properties of the system as
opposed to a more classical Maxwell-Boltzmann approach.

Problem 1 from Qualifying exam 2007-Fall.

Thermal and free expansion

One mole of an ideal gas undergoes a reversible thermal expansion from volume
V to volume 2V .
(a) What is the change in entropy of the gas?
(b) What is the change in entropy of the universe?
Now suppose the same expansion takes place as a free expansion.
(c) What is the change in entropy of the gas?
(d) What is the change in entropy of the universe?
Solution
(a) Since the process is isothermal, the internal energy does not change. Thus the
work done during the expansion must be equal to the heat absorbed by the gas.
Z 2V
Z 2V
RT
Q = W =
P dV =
dV = RT ln 2.
(3)
V
V
V
The entropy change of the gas is then
S=

Q
= R ln 2.
T

(4)

(b) The entropy change of the heat bath the gas is coupled to when expanding is
Sbath = Q/T = R ln 2, and so the total entropy change of the universe is
zero,
Su = Sgas + Sbath = 0,

(5)

as it must be for a reversible process.


(c) We recall that free expansion is an irreversible process in which a gas expands
into an insulated evacuated chamber. While for real gases the temperature may
change in the process, for ideal gases free expansion takes place at a constant
temperature. This is easy to see: in an ideal gas the molecules do not interact
and their average kinetic energy remains 3/2 kT (for monoatomic gas). For a
free expansion there is again no change in the internal energy of the gas, so the
initial and final states of the gas are the same as in part (a). The change in
entropy of the gas is thus again
Sgas = R ln 2.

(6)

(d) For a free expansion the state of the bath does not change and so Sbath = 0.
Thus for free expansion the total change in entropy of the universe is
Su = Sgas + Sbath = R ln 2
reflecting that free expansion is not reversible.
Problem 5 from Qualifying exam 2007-Fall.

(7)

Entropy change

A certain amount of water of heat capacity C is at a temperature of 0 C. It is


placed in contact with a heat reservoir at 100 C and the two come into thermal
equilibrium.
(a) What is the entropy change of the universe?
(b) The process is now divided into two stages: first the water is brought into
contact with a heat reservoir at 50 C and comes into thermal equilibrium;
then it is placed in contact with the heat reservoir at 100 C. What is the
entropy change of the universe?
(c) If we were to continue this subdivision into an infinite number of heat baths,
what would the entropy change of the universe be?
Solution
(a) The entropy change of the water is
Z Tf
Z Tf
CdT
Ti
dQ
=
= C ln
Sw =
T
T
T
f
Ti
Ti

(8)

where Ti = 273K and Tf = 373K, so


Sw = C ln

373
.
273

(9)

The entropy change of the reservoir is


Sr =

Q
TR Ti
Ti
= C
= C(1
)
TR
TR
TR

(10)

where Ti = 273K and TR = 373K. Thus the total entropy change of the universe
is
S = (ln

373
273
+
1)C = 0.044C
273
373

(11)

373
323
+
1)C = 0.010C.
323
373

(12)

(b) and after the second stage


S2 = (ln

Thus the total entropy change of the universe is


S = S1 + S2 = 0.023C
which is less than that found in part (a)(as we should expect).
(c) In this limit all heat transfer would be adiabatic and S = 0.

Problem 4 from Qualifying exam 2007-spring.

(13)

Spins in magnet

A magnet consists of N >> 1 noninteracting spins which can be in any of


three states: =1, 0, and -1. Each of these spins is described by a Hamiltonian:
E0 2 0 H

(14)

where H is the applied magnetic field.


(a) Write the partition function for this system.
(b) Calculate the Helmholtz free energy
(c) Calculate the entropy per spin, S/N in the limit as T .
(d) Does the specific heat per spin vanish, or tend to a constant as T ?
Explain your answer.
Solution
(a)
Z=


N
exp((E0 2 0 H)) = e(E0 0 H) + e(E0 +0 H) + 1

(15)

(b)


F = kT ln Z = N kT ln e(E0 0 H) + e(E0 +0 H) + 1

(16)

(c) As T , the probability of each state being occupied is roughly equal. Therefore, S/N = k ln 3. One can also solve this more explicitly by calculating
S = dF/dT , and then the limit.
(d) C = T dS/dT (constant V ). Since the entropy/spin approaches a constant as the
temperature approaches infinity, this means that dS/dT per spin is zero, and
the specific heat must be zero.

Problem 11 from Qualifying exam 2008-Fall.

Magnons

Spin waves (magnons) in a ferromagnetic solid have a dispersion relation of


the form:
2
= A|k|
Treating the magnons as elementary excitations, calculate the temperature dependence of the heat capacity of the spin system at low temperatures. You
might find the following integral to be useful:
Z

x3/2 dx
= finiteconstant
ex 1

Solution
The magnons are integer spin particles, but the number is not conserved. Therefore,
we will use the Planck distribution to describe them:
V
E=
(2)3

V 4
h

(4k2 dk) =
eh 1
(2)3

k2 dk
eh 1

0
2

Change of variable: let x = A


hk . We will then have that: dx = 2A
hkdk. Our
integral then becomes:
E=

V 4
hA

(2)3

1
5/2

R 
0

R
0

x
A
h

2

x3/2
dx
ex 1

1
dx 1
ex 1 2A
h k

V 4
hA

(2)3

R 
0

x
A
h

5/2

The heat capacity is therefore:


C=

dE
d 5/2

T
T 3/2
dT
dT

Problem 5 from Qualifying exam 2008-Spring.

2

1
dx
ex 1 2A
h

x
A
h

1/2

Hurricane

Some people believe that opening the windows of a house during a hurricane
could reduce the risk of having its roof blown off. Examine the possibilities
discussed below.
(a) Hurricane is a massive area of low pressure. The pressure in the center of
the storm is below 920 mbar1 . What would be the vertical component of
the force on the roof of a 10 by 10 meter house if the pressure inside the
house is 1013 mbar? Would this force lift a 10 ton roof? Would opening
of the windows help in this case and how realistic is this assumption?
(b) A typical wind speed of the category five storm is 70 m/s. Imagine that
the house has opened windows and the flow of air is passing below and
above the cylindrically shaped roof, see Fig. 1. What is the lifting force
on the roof in this situation 2 .

Figure 1: Airflow through and above the house.

Solution
(a) The pressure difference is 9.33 kPa. Which corresponds to a lifting force of 933
kN. A force required to lift a 10 ton roof is F = mg =100 kN. Thus, this pressure
difference will result in the roof being blown off. However, the assumption that
the pressure inside stays unchanged is unrealistic: hurricane is a relatively slow
moving storm, it takes hours for the pressure to drop and even a small air
exchange would equilibrate the inside-outside static pressure.
1 92

kPa
may use the following constants: molar mass of air is m = 29 g/mol, gas constant
R = Na k = 8.3 J/(K mol)
2 You

(b) We can estimate the pressure below and above the roof using the Bernoullis law
p2 p1 =

2
(v1 v22 ),
2

(17)

here we use subscript 1 for outside of the house and 2 for the inside. To estimate
the density one can use the ideal gas law
=

Pm
= 1.3kg/m3 ,
RT

(18)

or to recall that a molar volume is 22 dm3 /mol. The distance over the roof
of the house is /2 times longer than the distance directly through the house
which means that the speed of the flow is v1 = v2 /2, therefore


v 2 2
p2 p1 = 2
1 4.6kPa.
(19)
2
4
The resulting in this case force 460 kN is sufficient to lift the roof. Unlike in
part (a), this scenario is much more realistic.

Harmonic oscillator
Consider a one-dimensional quantum harmonic oscillator with energy levels
En = (n + 1/2)h

(a) What is the average energy of this oscillator at temperature T ?


(b) What is the specific heat of this oscillator?
(c) Give expressions for the specific heat appropriate for high temperature
(kB T 
h) and low temperature (kB T  h) limits.
Solution
(a) The average energy is
hEi =

P
En
d
n En e
P
=
ln Z
En
d
ne

where Z is the partition function


1

Z=

eh(n+1/2) =

Thus
hEi =

e 2 h
1
=
1 eh
2 sinh
h/2

d
ln(2 sinh
h/2) =
coth
h/2
d
2

(b)
C(T ) =

dhEi
1
1
=
dT
kB T 2 sinh2
h/2

(c) For kB T 
h we can make the approximation sinh2
h/2 ' (
h/2)2 and so
in this limit
C(T ) ' kB
For kB T 
h then sinh2
h/2 ' 41 eh and

C(T ) ' kB

h
kB T

Problem 9 from Qualifying exam 2008-Spring.

2

eh/kB T

Rubber band

A simple model of a rubber band is a one-dimensional (horizontal) chain consisting of N (N  1) linked segments, as shown in the diagram. Each segment
has just two possible states: horizontal with length a and vertical, contributing
nothing to the length. The segments are linked such that they cannot come
apart. The chain is in thermal contact with a reservoir at temperature T .

(a) If there is no energy difference between the two states, what is the average
length of the chain?
(b) The chain is now fixed at one end and a weight hung from the other end,
supplying a force F = M g as shown below. Determine the average length
of the chain at any temperature T . Find the length in the limits of low
and high T .

(c) In which temperature limit is the extension proportional to F (Hookes


law)? Calculate the constant of proportionality.
Solution
Z=

eEj

(20)

{j }

(a)
1
(1 j )a
2
X
N
L0 =
hlj i = a
2
j

Ej = 0 ; lj =

Problem 9 from Qualifying exam 2009-Fall.

(21)
(22)

(b)
Ej

hlj i

=
=
L

a
0 M g (1 j )
2
P
M g a
(1j ) 1
2
(1 j )a
j =1 e
2
P
M g a
(1
j)
2
j =1 e
2( a2 cosh

M ga
2T

a
sinh M2Tga )
2
M ga
2T

2 cosh
M ga
a
(1 + tanh
)
2
2T
N
X
M ga
M ga
Na
(1 + tanh
) = L0 (1 + tanh
)
hlj i =
2
2T
2T
j=1

(c) Hookes law is valid for

M ga
2T

1T 

1
2kB

M ga.

(23)
(24)

(25)
(26)
(27)

10

Water molecule

A water molecule can vibrate in the flexing mode in which the hydrogen
atoms move towards and away from each other without stretching the HO bond.
The oscillation of this mode is approximately harmonic with a frequency of
5.0 1013 Hz. What is the probability of a water molecule in its flexing ground
state and in the first excited state? Assume that the water is in equilibrium at
room temperature. The Boltzmann constant k is 1.38 1023 J/K and Plancks
constant h is 6.63 1034 J s.
Solution
The energy levels of a harmonic oscillator are given by En = (n+1/2)hf , where f is the
P
frequency and n = 0, 1, 2, 3, .... So the partition function is Z = n e(n+1/2)hf =
1 hf

e 2
1ehf

= 0.018. At T = 300K, hf /kT = 8.01. The probability of being in first

excited state is P1 = e(3/2)hf /kT /Z = 3.33 104 . The probability in being ground
state is e(1/2)hf /kT /Z = .9997.

Problem 6 from Qualifying exam 2010-Fall.

Você também pode gostar